Prove that K is a tensor using quotient theorem

  • #1
MatinSAR
528
174
Homework Statement
Prove that ##K_{ij}## is a tensor using quotient theorem.
Relevant Equations
Tensor analysis and quotient theorem.
1705965960948.png

$$K'_{ij}A'_{jk}=B'_{ik}=a_{ip}a_{kq}B_{pq}=a_{ip}a_{kq}K_{pr}A_{rq}=a_{ip}a_{kq}K_{pr}a_{jr}a_{kq}A'_{jk}$$$$K'_{ij}=a_{ip}a_{kq}a_{kq}a_{jr}K_{pr}$$

Can someone point out my mistake? What I've found shows that K is not a tensor.
It is different from my book and I cannot find my mistake. According to book K should be a 2nd-rank tensor.
 
Physics news on Phys.org
  • #2
You use k for a dummy/summation index on the right, but it is already used as a free index on the left.
 
  • Like
Likes MatinSAR and PhDeezNutz
  • #3
Hill said:
You use k for a dummy/summation index on the right, but it is already used as a free index on the left.
In last step I want to write ##A_{rq}## in prime corrdinates. So I had to use "k" again ... I wanted to prove using this method which is used by the book. Does this method work for my question in post #1?
1705987898773.png
1705987927299.png
 
  • #4
MatinSAR said:
So I had to use "k" again
But you can't. "k" is fixed by the left side of the equation, so you can't "sum for all k" on the right side of the equation.
MatinSAR said:
Does this method work for my question in post #1?
I'd rather convert the ##K_{ij}A_{jk}=B_{ik}## into a ##X_{ijk}C_{j}=B_{ik}## and apply the quotient rule you already have.
 
  • Like
Likes MatinSAR
  • #5
Hill said:
But you can't. "k" is fixed by the left side of the equation, so you can't "sum for all k" on the right side of the equation.

I'd rather convert the ##K_{ij}A_{jk}=B_{ik}## into a ##X_{ijk}C_{j}=B_{ik}## and apply the quotient rule you already have.
Then I will find out what is ##X##. But questions asks for ##K##.
 
  • #6
$$X'_{ijk}C'_j=B'_{ik}=a_{ip}a_{kq}B_{pq}=a_{ip}a_{kq}X_{prq}C_r$$ $$X'_{ijk}C'_j=a_{ip}a_{kq}X_{prq}a_{jr}C'_j$$$$X'_{ijk}=a_{ip}a_{jr}a_{kq}X_{prq}$$

I've proved it's a 3rd-rank tensor. But still I don't see how it helps me to prove that ##K## is a 2nd-rank tensor.
 
  • #7
MatinSAR said:
$$X'_{ijk}C'_j=B'_{ik}=a_{ip}a_{kq}B_{pq}=a_{ip}a_{kq}X_{prq}C_r$$ $$X'_{ijk}C'_j=a_{ip}a_{kq}X_{prq}a_{jr}C'_j$$$$X'_{ijk}=a_{ip}a_{jr}a_{kq}X_{prq}$$

I've proved it's a 3rd-rank tensor. But still I don't see how it helps me to prove that ##K## is a 2nd-rank tensor.
Very well. Now, back to ##K_{ij}A_{jk}=B_{ik}##.
Since ##A_{jk}## is arbitrary, we can take it as ##A_{jk}=C_j D_k##. Then we have ##K_{ij}C_j D_k=B_{ik}##.
Let ##X_{ijk}=K_{ij}D_k##. Then ##X_{ijk}C_j=B_{ik}##. You've shown that it makes ##X_{ijk}## a tensor.
So, now you have ##K_{ij}D_k=X_{ijk}##. Show that ##K_{ij}## is a tensor.
 
  • Like
Likes MatinSAR
  • #8
Hill said:
Very well. Now, back to ##K_{ij}A_{jk}=B_{ik}##.
Since ##A_{jk}## is arbitrary, we can take it as ##A_{jk}=C_j D_k##. Then we have ##K_{ij}C_j D_k=B_{ik}##.
Let ##X_{ijk}=K_{ij}D_k##. Then ##X_{ijk}C_j=B_{ik}##. You've shown that it makes ##X_{ijk}## a tensor.
So, now you have ##K_{ij}D_k=X_{ijk}##. Show that ##K_{ij}## is a tensor.
Great! What a clever idea! Thanks a lot for your help @Hill ...

I'll try. I hope I will be able to prove.
 
  • #9
@Hill
Sadly I wasn't able to prove for the same reason which was mentioned is post #1.
$$K'_{ij}D'_k=X'_{ijk}=a_{ip}a_{jq}a_{kr}X_{pqr}=a_{ip}a_{jq}a_{kr}K_{pq}D_{r}=a_{ip}a_{jq}a_{kr}K_{pq}a_{kr}D'_k$$$$K'_{ij}=a_{ip}a_{jq}a_{kr}a_{kr}K_{pq}$$

I am trying to use my book's method. This method worked for equation in post #6 but It doesn't work for equation in post #1 and it doesn't work in this post as well ... I dont know why !
 
  • Like
Likes PhDeezNutz
  • #10
MatinSAR said:
@Hill
Sadly I wasn't able to prove for the same reason which was mentioned is post #1.
$$K'_{ij}D'_k=X'_{ijk}=a_{ip}a_{jq}a_{kr}X_{pqr}=a_{ip}a_{jq}a_{kr}K_{pq}D_{r}=a_{ip}a_{jq}a_{kr}K_{pq}a_{kr}D'_k$$$$K'_{ij}=a_{ip}a_{jq}a_{kr}a_{kr}K_{pq}$$

I am trying to use my book's method. This method worked for equation in post #6 but It doesn't work for equation in post #1 and it doesn't work in this post as well ... I dont know why !
##K'_{ij}D'_k=X'_{ijk}=a_{ip}a_{jq}a_{kr}X_{pqr}=a_{ip}a_{jq}a_{kr}K_{pq}D_{r}=a_{ip}a_{jq}K_{pq}a_{kr}D_{r}=a_{ip}a_{jq}K_{pq}D'_k##
 
  • Like
Likes PhDeezNutz and MatinSAR
  • #11
Hill said:
##K'_{ij}D'_k=X'_{ijk}=a_{ip}a_{jq}a_{kr}X_{pqr}=a_{ip}a_{jq}a_{kr}K_{pq}D_{r}=a_{ip}a_{jq}K_{pq}a_{kr}D_{r}=a_{ip}a_{jq}K_{pq}D'_k##
I've found my mistake. Thanks for your help @Hill ;)
 
  • Like
Likes PhDeezNutz and Hill

1. How do you prove that K is a tensor using quotient theorem?

To prove that K is a tensor using quotient theorem, we need to show that K satisfies the transformation properties of a tensor under coordinate transformations.

2. What is the quotient theorem in tensor analysis?

The quotient theorem in tensor analysis states that if a quantity K transforms like a tensor under coordinate transformations, then it is a tensor.

3. What are the steps involved in proving that K is a tensor using quotient theorem?

The steps involved in proving that K is a tensor using quotient theorem include showing that K transforms according to the tensor transformation law under coordinate transformations.

4. Why is it important to prove that K is a tensor using quotient theorem?

Proving that K is a tensor using quotient theorem is important because it ensures that K behaves like a tensor and obeys the rules of tensor analysis, making it a valid quantity in the context of the problem being studied.

5. Can you provide an example of proving that K is a tensor using quotient theorem?

Sure, one example of proving that K is a tensor using quotient theorem is by showing that K transforms as a rank-2 tensor under coordinate transformations, satisfying the tensor transformation law.

Similar threads

  • Advanced Physics Homework Help
Replies
1
Views
1K
  • Advanced Physics Homework Help
Replies
1
Views
2K
  • Advanced Physics Homework Help
Replies
6
Views
2K
  • Linear and Abstract Algebra
Replies
3
Views
1K
  • Classical Physics
Replies
30
Views
2K
  • Calculus and Beyond Homework Help
Replies
1
Views
1K
  • Special and General Relativity
Replies
7
Views
2K
  • Linear and Abstract Algebra
Replies
1
Views
3K
  • Topology and Analysis
Replies
9
Views
2K
  • Calculus and Beyond Homework Help
Replies
10
Views
2K
Back
Top